0

Vectors from a geometric viewpoint - class-XI

Description: vectors from a geometric viewpoint
Number of Questions: 29
Created by:
Tags: applications of vector algebra addition of vectors vector vector algebra maths vectors and transformations vectors vectors, lines and planes vector algebra - i
Attempted 0/29 Correct 0 Score 0

If $\overrightarrow A ,\overrightarrow B $ and $\overrightarrow C $ are vectors such that $\left| {\overrightarrow B } \right| = \left| {\overrightarrow C } \right|$ , then  $\left{ {\left( {\overrightarrow A  + \overrightarrow B } \right)} \right. \times \left. {\left( {\overrightarrow A  + \overrightarrow C } \right)} \right} \times \left( {\overrightarrow B  \times \overrightarrow C } \right).\left( {\overrightarrow B  + \overrightarrow C } \right) = 1 $  these relation is ?

  1. True

  2. False


Correct Option: B

If the vectors $\overrightarrow a  = \left( {2,{{\log } _3}x,\;a} \right)$ $and\;\overrightarrow b  = \left( { - 3,a{{\log } _3}x,{{\log } _3}x} \right)$ are included at an acute angle then-

  1. a=0

  2. a<0

  3. a>0

  4. None of these


Correct Option: C
Explanation:

$\begin{array}{l} \overrightarrow { a } =2\left( { { { \log   } _{ 3 } }x,a } \right) ,\overrightarrow { b } =\left( { -3,a{ { \log   } _{ 3 } }x,{ { \log   } _{ 3 } }x } \right)  \ \overrightarrow { a } \overrightarrow { b } =\left| a \right| \left| b \right| \cos  \theta  \ \cos  \theta =\frac { { \overrightarrow { a } .\overrightarrow { b }  } }{ { \left| a \right| \left| b \right|  } }  \ =\frac { { -6+a{ { \left( { { { \log   } _{ 3 } }x } \right)  }^{ 2 } }+a.{ { \log   } _{ 3 } }x } }{ { \sqrt { 4+{ { \left( { { { \log   } _{ 3 } }x } \right)  }^{ 2 } }+{ a^{ 2 } } } .\sqrt { 9+{ a^{ 2 } }{ { \left( { { { \log   } _{ 3 } }x } \right)  }^{ 2 } }+{ { \left( { { { \log   } _{ 3 } }x } \right)  }^{ 2 } } }  } }  \ for\, \, acute\, \, angle\, \, a>0 \ Option\, \, \, C\, \, is\, \, correct. \end{array}$

If $\displaystyle a\times b=a\times c,a\neq 0,$ then

  1. $\displaystyle b=c+\lambda a$

  2. $\displaystyle c=a+\lambda b$

  3. $\displaystyle a=b+\lambda c$

  4. None of these


Correct Option: A
Explanation:

$\displaystyle a\times b=a\times c\Rightarrow a\times b-a\times c=0$
$\Rightarrow a\times (b-c)=0$ $\Rightarrow a \parallel (b-c)$
$\Rightarrow b-c = \lambda a\Rightarrow b= c+\lambda a$

$\displaystyle a\times \left ( b+c \right )+b\times \left ( c+a \right )+c\times \left ( a+b \right )$ is equal to

  1. $\displaystyle 2\left [ a:b:c \right ]$

  2. $0$

  3. $3$

  4. None of these


Correct Option: B
Explanation:

$\displaystyle a\times \left ( b+c \right )+b\times \left ( c+a \right )+c\times \left ( a+b \right )$
$=a\times b+a\times c+b\times c+b\times a+c\times a+c\times b$
$=a\times b-c\times a+b\times c-a\times b+c\times a-b\times c = 0$

Let $\displaystyle a=i+j$ and $\displaystyle b=2i-k,$ the point of intersection of the lines $\displaystyle r\times a=b\times a $ and $\displaystyle r\times b=a\times b $ is

  1. $\displaystyle -i+j+k$

  2. $\displaystyle 3i-j+k$

  3. $\displaystyle 3i+j-k$

  4. $\displaystyle i-j-k$


Correct Option: C
Explanation:

$\vec r \times \vec a = \vec b \times \vec a\Rightarrow \vec{r} = \vec{b}+\lambda \vec{a}$
and $\vec r \times \vec b = \vec a \times \vec b\Rightarrow \vec{r} = \vec{a}+\mu \vec{b}$
For intersection of both the lines, $\vec{b}+\lambda \vec{a}=\vec{a}+\mu \vec{b}$
Comparing coefficients, $\lambda=\mu = 1$
Hence point of intersection is $\vec{r}=\vec{a}+\vec{b} = 3\hat{i}+\hat{j}-\hat{k}$

If $\overline{a},\overline{b},\overline{c}$ are three non-zero vectors and $\overline{a}\neq\overline{b}$, $\overline{a}\times\overline{c}=\overline{b}\times\overline{c}$, then

  1. $\overline{a}-\overline{b}$ is parallel to $\overline{c}$

  2. $\overline{a}-\overline{b}$ is perpendicular to $\overline{c}$

  3. $\overline{a}+\overline{b}$ is parallel to $\overline{c}$

  4. $\overline{a}+\overline{b}$ is perpendicular to $\overline{c}$


Correct Option: A
Explanation:

We get 

$(\vec{a}-\vec{b})\times\vec{c}=0$
So    $(\vec{a}-\vec{b})\parallel \vec{c}$

If $a +2b +3c = 0$, then $a \times b + b\times c + c\times a = ka\times b,$
Where $k$ is equal to ?

  1. $0$

  2. $1$

  3. $2$

  4. $3$


Correct Option: C
Explanation:

Given,

$\begin{array}{l} a+2b+3c=0 \ a=-\left( { 2b+3c } \right)  \ a\times a=-\left( { 2\left( { b\times a } \right) +3\left( { c\times a } \right)  } \right)  \ \Rightarrow 0=-\left( { 2b\times a+3\left( { c\times a } \right)  } \right) ..........\left( i \right)  \end{array}$
Again,
$\begin{array}{l} 2b=-\left( { a+3c } \right)  \ 2b\times b=-\left( { a\times b+3c\times b } \right)  \ \Rightarrow 0=-\left( { a\times b+3c\times b } \right) ..........\left( { ii } \right)  \end{array}$
equation (i) and (ii)
$\begin{array}{l} -\left( { 2b\times a+3c\times a } \right) +\left( { a\times b } \right) +3c\times b=0 \ \Rightarrow 3a\times b-3c\times a-3b\times c=0 \end{array}$
We know that,
$a \times b =  - b \times a$
$a \times b = c \times a + b \times c...............\left( {iii} \right)$
Now,$ATQ;$
$a \times b + b \times c + c \times a = K\,\,\,a \times b$
$1 = 2a \times b = k\,a \times b$         {from equation (iii)}
$\therefore k=2$
Option $C$ is correct answer.


If $\left| \vec { a }  \right| =1,\ \left| \vec { b }  \right| =2,\ (\vec { a },\vec { b })=\dfrac{2\pi}{3}$ then $\left{(\vec { a } +3\vec { b } )\times \left( 3\vec { a } -\vec { b }  \right) \right}^{2}=$


  1. $425$

  2. $\dfrac{147}{2}$

  3. $325$

  4. $300$


Correct Option: B
Explanation:

$|\bar{a}|=1, |\bar{b}|=2, (\bar{a}, \bar{b})=\dfrac{2z}{3}$


$[|\bar{a}|+3|\bar{b}|]=1+3\times 2=1+6=7$ ......... $(1)$


Also, $3|\bar{a}|-|\bar{b}|=3\times 1-2=3-2=1$ ......... $(2)$

$|(\bar{a}+3\bar{b})\times (3\bar{a}-\bar{b})|=|(\bar{a}+3\bar{b})||3\bar{a}-\bar{b}|\sin \theta$

$=|\bar{a}+3\bar{b}||3\bar{a}-\bar{b}|\sin (\bar{a}, \bar{b})$

$=|(\bar{a}+3\bar{b}||3\bar{a}-\bar{b}|\sin\left(\dfrac{2z}{3}\right)$

$=|\bar{a}+3\bar{b}||3\bar{a}-\bar{b}|\sin 60^{o}$

$|\bar{a}+3\bar{b}||3\bar{a}-\bar{b}|\left(\dfrac{\sqrt{3}}{2}\right)$

from $(1)$ & $(2)$

$=7\times 1\dfrac{\surd{3}}{2}=\dfrac{7\sqrt{3}}{2}$

$[(\bar{a}+3\bar{b}\times (3\bar{a}-\bar{b})]^{2}=\left(7\dfrac{\sqrt{3}}{2}\right)^{2}$

$=\dfrac{49\times 3}{4}=\dfrac{147}{2}$

If $\vec a = \hat i + \hat j + \hat k,\,\vec b = \hat i + \hat j,\,\,\hat c = \hat i$ and $\left( {\vec a \times \vec b} \right) \times \vec c = \lambda \vec a \times \mu \vec b$ then $\lambda  + \mu $

  1. $0$

  2. $1$

  3. $2$

  4. $3$


Correct Option: A
Explanation:
Few things to consider 
$\hat i\times \hat i = \hat j\times \hat j=\hat k\times  \hat k =0$
$\hat i\times \hat j = \hat k , \hat j \times \hat k=\hat i , \hat k \times \hat i =\hat k$
$\hat j\times \hat i = -\hat k , \hat i \times \hat k=-\hat j , \hat k \times \hat j =-\hat i$
Now, $(\vec a \times \vec b) \times \vec c \Rightarrow $
$\vec a =(\hat i+\hat j+\hat k), \vec b=\hat i+\hat j, \vec c=\hat i$
$\Rightarrow \ (\hat i+\hat j+\hat k) \times (\hat i+\hat j)\times (\hat i)$
$\Rightarrow \ (\hat k-\hat k+\hat j-\hat i)\times (\hat i)$
$\Rightarrow \ -\hat k$
Which does not involve any term like $\lambda \vec a\times \mu \vec b \ \therefore \ \lambda +\mu =0$


Let $\vec{a} = \widehat{i} + \widehat{j}$, $\vec{b} = 2 \widehat{i} - \widehat{k}$, then vector $\vec{r}$ satisfying the equations $\vec{r} \times \vec{a} = \vec{b} \times \vec{a}$ and $\vec{r} \times \vec{b} = \vec{a} \times \vec{b}$ is

  1. $\widehat{i} - \widehat{j} + \widehat{k}$

  2. $3\widehat{i} - \widehat{j} + \widehat{k}$

  3. $3\widehat{i} + \widehat{j} - \widehat{k}$

  4. $\widehat{i} - \widehat{j} - \widehat{k}$


Correct Option: C
Explanation:

$\vec{r} \times \vec{a} = \vec{b} \times \vec{a}$ or $(\vec{r} - \vec{b}) \times \vec{a} = 0$
$\vec{r} \times \vec{b} = \vec{a} \times \vec{b}$ or $(\vec{r} - \vec{a}) \times \vec{b} = 0$
If $\vec{r} = x \widehat{i} + y \widehat{j} + z\widehat{k}$, then 


$\begin{vmatrix}\widehat{i} & \widehat{j} & \widehat{k}\ x-2 & y & z+1\ 1 & 1 & 0\end{vmatrix} = 0$ and $\begin{vmatrix}\widehat{i} & \widehat{j} & \widehat{k}\ x-1 & y-1 & z\ 2 & 0 & -1 \end{vmatrix}= 0$

$\Rightarrow z + 1 = 0, x - y = 2$
and $y -1 = 0, x - 1 + 2z = 0$
$\Rightarrow x = 3, y = 1, z = -1$

If the vector $\bar{c}, \bar{a} = x\bar{i}+y\bar{j}+ z\bar{k}, \bar{b}= \bar{j}$ are such that $\bar{a}, \bar{c}, \bar{b}$ from R.H.S then $\bar{c}$ = 

  1. $z\bar{i} -x\bar{k}$

  2. $z\bar{i} -3\bar{k}$

  3. $x\bar{j} -y\bar{k}$

  4. $y\bar{j} -x\bar{k}$


Correct Option: A

If $a,b,c$ are unit vectors, then the maximum value of $|a+2b|^{2}+|b+3c|^{2}+|c+4a|^{2}$ is 

  1. $50$

  2. $21$

  3. $48$

  4. $58$


Correct Option: A
Explanation:
Given,

$\left|a+2b\right|^2+\left|b+3c\right|^2+\left|c+4a\right|^2$

$=\left(a+2b\right)^2+\left(b+3c\right)^2+\left(4a+c\right)^2$

$=a^2+4ab+4b^2+b^2+6bc+9c^2+c^2+8ac+16a^2$

$=17a^2+4ab+8ac+5b^2+10c^2+6bc$

$=50$

If $\displaystyle \bar{a}+p\bar{b}+q\bar{c}=0 $ then

  1. $\displaystyle p(\bar{a}\times\bar{b})=pq(\bar{b}\times\bar{c})=q(\bar{c}\times\bar{a})$

  2. $\displaystyle \bar{a}\times\bar{b}=pq(\bar{c}\times\bar{a})$

  3. $\displaystyle \bar{c}\times\bar{a}=p(\bar{a}\times\bar{b})$

  4. $\displaystyle \bar{a}\times\bar{c}=q(\bar{b}\times\bar{c})$


Correct Option: A
Explanation:

$\bar{a}+p\bar{b}+q\bar{c}=0$
$\bar{b}=\dfrac{-q\bar{c}-\bar{a}}{p}$

Then
$p(\bar{a}\times b)$
$=p(\bar{a}\times (\dfrac{-q\bar{c}-\bar{a}}{p}))$

$=((q\bar{c}+\bar{a})\times \bar{a})$
$=q(\bar{c}\times \bar{a})$
$=q(\bar{c}\times (-p\bar{b}-q\bar{c}))$
$=q((p\bar{b}+q\bar{c})\times \bar{c})$
$=pq(\bar{b}\times \bar{c})$.

If the vector $a, b$ and $c$ form the sides $BC, CA $ and $AB $ and equal magnitute respectively of a triangle $ABC,$ then

  1. $ a \cdot b + b\cdot c + c \cdot a = 0$

  2. $a \times b = b \times c = c \times a$

  3. $a \cdot b = b\cdot c = c \cdot a$

  4. $a \times b + b \times c + c \times a = O$


Correct Option: B
Explanation:

By triangle law, $\overrightarrow { a } +\overrightarrow { b } +\overrightarrow { c } =\overrightarrow { 0 } $
Taking cross product by $\overrightarrow { a } ,\overrightarrow { b } ,\overrightarrow { c } $ respectively 
$\overrightarrow { a } \times \left( \overrightarrow { a } +\overrightarrow { b } +\overrightarrow { c }  \right) =\overrightarrow { a } \times \overrightarrow { 0 } =\overrightarrow { 0 } $
$\Rightarrow \overrightarrow { a } \times \overrightarrow { a } \times \overrightarrow { a } \times \overrightarrow { b } +\overrightarrow { a } \times \overrightarrow { c } =\overrightarrow { a } $
$\Rightarrow \overrightarrow { a } \times \overrightarrow { b } =\overrightarrow { c } \times \overrightarrow { a } \quad \left[ \because \overrightarrow { a } \times \overrightarrow { a } =\overrightarrow { 0 }  \right] $
Similarly, $\overrightarrow { a } \times \overrightarrow { b } =\overrightarrow { b } \times \overrightarrow { c. } $
$\therefore \overrightarrow { a } \times \overrightarrow { b } =\overrightarrow { b } \times \overrightarrow { c } =\overrightarrow { c } \times \overrightarrow { a. } $

If $\displaystyle a\cdot b=a\cdot c$ and $\displaystyle a\times b=a\times c,$ then

  1. either $\displaystyle a=0$ or $\displaystyle b=c$

  2. $a$ is parallel to $\displaystyle \left ( b-c \right )$

  3. $a$ is perpendicular to $\displaystyle \left ( b-c \right )$

  4. None of these


Correct Option: A
Explanation:

Given,  $\displaystyle a\cdot b=a\cdot c$ and $\displaystyle a\times b=a\times c,$
$\Rightarrow a\cdot (b-c) = 0$ and $a\times (b-c) = 0$
Hence  either $a=0$ or $b=c$

Let $\displaystyle \vec{a}=\hat{i}+\hat{j}$ & $\displaystyle \vec{b}=2\hat{i}+\hat{j}$ The point of intersection of the lines $\displaystyle \vec{r}\times \vec{a}=\vec{b}\times \vec{a}&amp; \vec{r}\times \vec{b}=\vec{a}\times \vec{b}$ is

  1. $\displaystyle -\hat{i}+\hat{j}+\hat{k}$

  2. $\displaystyle -3\hat{i}-\hat{j}+\hat{k}$

  3. $\displaystyle 3\hat{i}+\hat{j}-\hat{k}$

  4. $\displaystyle \hat{i}-\hat{j}-\hat{k}$


Correct Option: C
Explanation:

$\displaystyle \overrightarrow{a}=\hat{i}+\hat{j}$ & $\displaystyle \hat{b}=\hat{2i}-\hat{k}:\overrightarrow{r}\times \overrightarrow{a}=\overrightarrow{b}\times \overrightarrow{a}\Rightarrow \left ( \overrightarrow{r}-\overrightarrow{b} \right )\times \overrightarrow{a}=0
\Rightarrow \overrightarrow{r}=\overrightarrow{b}+\lambda \overrightarrow{a}$ 

Similarly $\displaystyle \overrightarrow{r}\times \overrightarrow{b}=\overrightarrow{a}\times \overrightarrow{b}\Rightarrow \overrightarrow{r}=\overrightarrow{a}+\mu \overrightarrow{b}$ 
Substitute the vector $\displaystyle \overrightarrow{a}$ & $\displaystyle \overrightarrow{b}$ in (i) & (ii) and equating we get 
$\displaystyle 2\hat{i}-\hat{k}+\lambda \left ( \hat{i}+\hat{j} \right )=\hat{i}+\hat{j}+\mu \left ( 2\hat{i}-\hat{k} \right )
\Rightarrow 2+\lambda =1+2\mu ,\lambda =1,\mu =1
\therefore $ Point of intersection is $\displaystyle 3\hat{i}+\hat{j}-\hat{k}$

Let $\displaystyle \vec{A}=2\vec{i}+\vec{k},\,\vec{B}=\vec{i}+\vec{j}+\vec{k},$ and $\displaystyle \vec{C}=4\vec{i}-3\vec{j}+7\vec{k}$ Determine a vector $\displaystyle \vec{R}$satisfying $\displaystyle \vec{R}\times \vec{B}=\vec{C}\times \vec{B}$ and $\displaystyle \vec{R}.\vec{A}=0$

  1. $\displaystyle -\hat{i}-8\hat{j}+2\hat{k}$

  2. $\displaystyle -8\hat{i}-\hat{j}+2\hat{k}$

  3. $\displaystyle -2\hat{i}-\hat{j}+8\hat{k}$

  4. $\displaystyle -\hat{i}-2\hat{j}+8\hat{k}$


Correct Option: A
Explanation:

Given, $\displaystyle \vec{R}\times \vec{B}=\vec{C}\times \vec{B} \Rightarrow \vec{R}\times \vec{B}-\vec{C}\times \vec{B}=0$
$\Rightarrow (\vec{R}-\vec{C})\times \vec{B}=0\Rightarrow R = \vec{C}+k\vec{B}$
Also given, $\vec{R}\cdot \vec{A} =0\Rightarrow k =- \cfrac{\vec{C}\cdot \vec{A}}{\vec{B}\cdot\vec{A}}=-\cfrac{15}{3}=-5$
Hence $\vec{R} =\vec{C}-5\vec{B}=-\hat{i}-8\hat{j}+2\hat{k} $ 

Unit vector $\vec r$ which satisfies $\vec r \times \vec b = \vec r \times \vec c$ where $\vec b = \widehat i + 2 \widehat j + \widehat k $ & $ \vec c = 3 \widehat i + 2 \widehat k $, is

  1. $\displaystyle \pm \left ( \frac{2 \widehat i - 2 \widehat j + \widehat k}{3}\right )$

  2. $\displaystyle \pm \left ( \frac{2 \widehat i + 2 \widehat j + \widehat k}{3}\right )$

  3. $\displaystyle \pm \left ( \frac{\widehat i + \widehat j + \widehat k}{\sqrt 3}\right )$

  4. $\pm \widehat i$


Correct Option: A
Explanation:

Given $\vec b = \widehat i + 2 \widehat j + \widehat k $ & $ \vec c = 3 \widehat i + 2 \widehat k $
$\vec r \times \vec b = \vec r \times \vec c$
$(\vec r \times \vec b) - (\vec r \times \vec c) = \vec 0 $
$\Rightarrow \vec r \times (\vec b - \vec c) = \vec 0$
$\Rightarrow \vec r = \lambda (\vec b - \vec c) = \lambda (- 2 \widehat i + 2 \widehat j - \widehat k)$
$\Rightarrow \displaystyle \widehat r = \pm \left ( \frac{2 \widehat i - 2 \widehat j + \widehat k}{3} \right )$

Let $\vec a = \widehat i + \widehat j$ and $\vec b = 2 \widehat i - \widehat k$, then the point of intersection of lines $\vec r \times \vec a = \vec b \times \vec a$ and $\vec r \times \vec b = \vec a \times \vec b$ is

  1. $\widehat i + \widehat j + \widehat k$

  2. $3 \widehat i - \widehat j + \widehat k$

  3. $3\widehat i + \widehat j - \widehat k$

  4. $\widehat i - \widehat j-\widehat k$


Correct Option: C
Explanation:

$\vec r \times \vec a = \vec b \times \vec a\Rightarrow \vec{r} = \vec{b}+\lambda \vec{a}$
and $\vec r \times \vec b = \vec a \times \vec b\Rightarrow \vec{r} = \vec{a}+\mu \vec{b}$
For intersection of both the lines, $\vec{b}+\lambda \vec{a}=\vec{a}+\mu \vec{b}$
Comparing coefficients, $\lambda=\mu = 1$
Hence point of intersection is $\vec{r}=\vec{a}+\vec{b} = 3\hat{i}+\hat{j}-\hat{k}$

If $\overline{a}\times\overline{b}=\overline{b}\times\overline{c}$, then

  1. $\overline{b}=\overline{a}\times\overline{c}$

  2. $\overline{b}||\overline{a}-\overline{c}$

  3. $\overline{b}\Vert(\overline{a}+\overline{c})$

  4. $\overline{b}=\overline{a}-\overline{c}$


Correct Option: C
Explanation:

$\overrightarrow{a}\times\overrightarrow{b}=\overrightarrow{b}\times\overrightarrow{c}$
$(\overrightarrow{a}+\overrightarrow{c})\times\overrightarrow{b}=0$
$\Rightarrow (\overrightarrow{a}+\overrightarrow{c})\parallel \overrightarrow{b}$

If three vectors $\overline{a},\overline{b},\ \overline{c}$ are such that $\overline{a}\neq 0$, $\overline{a}\times\overline{b}=2\overline{a}\times\overline{c},\ |\overline{a}|=|\overline{c}|=1,\ |\overline{b}|=4$ and the angle between $|\overline{b}|$ and $|\overline{c}|$ is $\displaystyle \cos^{-1}\frac{1}{4}$, then $\overline{b}-2\overline{c}=\lambda\overline{a}$ where $\lambda$ is equal to

  1. $\pm 2$

  2. $\pm 4$

  3. $\displaystyle \dfrac{1}{2}$

  4. $\displaystyle \dfrac{1}{4}$


Correct Option: B

If $\vec{a}\times\vec{b}=\vec{c}\times\vec{d}$ and $\vec{a}\times\vec{c}=\vec{b}\times\vec{d}$, then

  1. $\vec{a}+\vec{b}=\vec{c}+\vec{d}$

  2. $\vec{a}-\vec{d}$ is parallel to $\vec{b}-\vec{c}$

  3. $\vec{a}-\vec{d}$ is perpendicular to $\vec{b}-\vec{c}$

  4. $\vec{a}-\vec{b}$ is perpendicular to $\vec{a}-\vec{b}$


Correct Option: B
Explanation:

$\vec{a}\times \vec{b}= \vec{c}\times \vec{d}$
$\vec{a}\times \vec{c}= \vec{b}\times \vec{d}$
$\vec{a}\times (\vec{b}-\vec{c})= (\vec{c}-\vec{b})\times \vec{d}$
$(\vec{a}-\vec{d})\times (\vec{b}-\vec{c})= 0$
$(\vec{a}-\vec{d})\ \parallel \ (\vec{b}-\vec{c})$

If $\vec {a},\vec {b},\ \vec {c}$ are non-zero non-collinear vectors such that $\vec {a}\times\vec {b}=\vec {b}\times\vec {c}=\vec {c}\times\vec {a}$ , then $\vec {a}+\vec {b}+\vec {c}=$

  1. $abc$

  2. $-1$

  3. $\vec {0}$

  4. $2$


Correct Option: C
Explanation:

$\vec{a}\times \vec{b}=\vec{b}\times \vec{c}$
$(\vec{a}+\vec{c})\times \vec{b}=o$
$(\vec{a+\vec{c}}+\vec{b})\times \vec{b}=\vec{b\times \vec{b}}$
$(\vec{a}+\vec{c}+\vec{b})\times \vec{b}=0$
$\vec{a}+\vec{c}+\vec{b}=\vec{0}$   hence $\vec{a},\vec{b},\vec{c}$ are non collinear

If $\vec {a}\times \vec {b}=\vec {c}\times \vec {d},\vec {a}\times \vec {c}=\vec {b}\times \vec {d}$, then

  1. $\vec {a}-\vec {d}$ is parallel to $\vec {b}-\vec {c}$

  2. $\vec {a}-\vec {b}$ is parallel to $\vec {c}-\vec {d}$

  3. $\vec {a}-\vec {c}$ is parallel to $\vec {b}-\vec {d}$

  4. $\vec {a}+\vec {b}$ is parallel to $\vec {c}+\vec {d}$


Correct Option: A
Explanation:

$\vec{a}\times \vec{b}=\vec{c}\times \vec{d}  -(1)$
$\vec{a}\times \vec{c}=\vec{b}\times \vec{d}  -(2)$
$(1) - (2)$
$\vec{a}\times \vec{b}+\vec{c}\times \vec{a}=\vec{c}\times \vec{d}+\vec{d}\times \vec{b}$
$(\vec{a}-\vec{d})\times \vec{b}=c\times (\vec{d}-\vec{a})$
$(\vec{a}-\vec{d})\times \vec{b}+(\vec{d}-\vec{a})\times \vec{c}=0$
$(\vec{a}-\vec{d})\times (\vec{b}-\vec{c})=0$
which mean $(\vec{a}-\vec{d})||(\vec{b}-\vec{c})$

If $\vec {a}$ and $\vec {b}$ are not perpendicular to each other and $\vec {r}\times\vec {a}=\vec {b}\times\vec {a},\ \vec {r}.\vec {c}=0$, then $\vec {r}$ is equal to

  1. $\vec {a}-\vec {c}$

  2. $\vec {b}+\lambda\vec {a}$, for all scalars $\lambda$

  3. $\displaystyle \vec {b}-\dfrac{(\vec {b}.\vec {c})}{(\vec {a}.\vec {c})}\vec {a}$

  4. $\vec {a}+\vec {c}$


Correct Option: C
Explanation:

Let $r,a,b$ and $c$ be vectors.
It is given that
$r\times a=b\times a$
$r\times a-(b\times a)=0$
$(r-b)\times a=0$
Hence $r-b$ is a vector parallel to vector $a$.
$r=b+\mu a$ ...(i)
It is given that $r.c=0$.
Hence $r$ vector is perpendicular to $c$ vector.
$(b+\mu a).c=0$ ...(from i)
$b.c+\mu(a.c)=0$
$\mu(a.c)=-b.c$
$\mu=-\dfrac{b.c}{a.c}$
Hence $r=b-\dfrac{b.c}{a.c}a$

If $a$ and $b$ are two unit vectors inclined at an angle $\dfrac { \pi  }{ 3 }$, then $\left{ a\times \left( b+a\times b \right)  \right} \cdot b$ is equal to

  1. $\dfrac { 1 }{ 4 } $

  2. $\dfrac { -3 }{ 4 } $

  3. $\dfrac { 3 }{ 4 } $

  4. $\dfrac { 1 }{ 2 } $


Correct Option: B
Explanation:

Given, $\left| a \right| =\left| b \right| =1$ and $a\cdot b=\cos { \dfrac { \pi  }{ 3 }  } $
Consider,
$\left{ a\times \left( b+a\times b \right)  \right} \cdot b=\left{ a\times b+a\times \left( a\times b \right)  \right} \cdot b$
            $=\left( a\times b \right) \cdot b+\left{ \left( a\cdot b \right) \cdot a-\left( a\cdot a \right) \cdot b \right} \cdot b$
            $=\left[ \begin{matrix} a & b & b \end{matrix} \right] +{ \left( a\cdot b \right)  }^{ 2 }-{ \left| a \right|  }^{ 2 }{ \left| b \right|  }^{ 2 }$
            $=0+\cos ^{ 2 }{ \dfrac { \pi  }{ 3 }  } -1=\dfrac { 1 }{ 4 } -1=\dfrac { -3 }{ 4 } $

Let $\vec{\lambda }=\vec{a}\times \left ( \vec{b}+\vec{c} \right )$, $\vec{\mu }=\vec{b}\times \left ( \vec{c}+\vec{a} \right )$ and $\vec{\nu }=\vec{c}\times \left ( \vec{a}+\vec{b} \right )$, then

  1. $\vec{\lambda }+\vec{\mu }=\vec{\nu }$

  2. $\vec{\lambda }, \vec{\mu }, \vec{\nu }$ are coplanar

  3. $\vec{\lambda }+\vec{\nu }=2\vec{\mu }$

  4. None of these


Correct Option: B
Explanation:

$\vec { \lambda  } +\vec { \mu  } =\vec { a } \times \left( \vec { b } +\vec { c }  \right) +\vec { b } \times \left( \vec { c } +\vec { a }  \right) $

$= \vec { a } \times \vec { b } +\vec { a } \times \vec { c } +\vec { b } \times \vec { c } +\vec { b } \times \vec { a }$
$ = \left( \vec { a } +\vec { b }  \right) \times \vec { c }$
$= -\vec { \nu  } \ \Rightarrow \quad \vec { \nu  }$
$ =-\left( \vec { \lambda  } +\vec { \mu  }  \right) $

one vector is expressed as linear combination of other two vectors
Hence,
$\vec { \lambda  } ,\vec { \mu  } ,\vec { \nu  } $ are coplanar vectors.

Let $\vec{r}\times \vec{a}=\vec{b}\times \vec{a}$ and $\vec{r}.\vec{c}=0$, where $\vec{a}\vec{b}\neq 0$, then $\vec{r}$ is equal to

  1. $\vec{b}+t\vec{a}$ where $t$ is a scalar

  2. $\displaystyle \vec{b}-\dfrac{\vec{b}.\vec{c}}{\vec{a}.\vec{c}}\vec{a}$

  3. $\vec{a}-\vec{c}$

  4. $None\ of\ these$


Correct Option: B
Explanation:

$\vec{r}\times \vec{a}=\vec{b}\times \vec{a}$

$\Rightarrow \vec{r}\times \vec{a}-\vec{b}\times \vec{a}=0$
$\Rightarrow (\vec{r}-\vec{b}\times \vec{a} = 0\Rightarrow \vec{r} = \vec{b}+t\vec{a}$

Now taking dot product with $\vec{c}$ both side,
$\Rightarrow \vec{r}\cdot\vec{c} = \vec{b}\cdot\vec{c}+t \vec{a}\cdot\vec{c}=0$
$\Rightarrow t = -\dfrac{\vec{b}\cdot\vec{c}}{\vec{a}\cdot\vec{c}}$

Hence $\vec{r} = \vec{b}-\dfrac{\vec{b}\cdot\vec{c}}{\vec{a}\cdot\vec{c}}\vec{a}$

If $\overrightarrow{a}, \overrightarrow{b}, \overrightarrow{c}$ are any three vectors in space then $\left ( \overrightarrow{c}+\overrightarrow{b} \right )\times \left ( \overrightarrow{c}+\overrightarrow{a} \right ).\left ( \overrightarrow{c}+\overrightarrow{b}+\overrightarrow{a} \right )$ is equal to

  1. $3\begin{bmatrix}

    \overrightarrow{a} & \overrightarrow{b} & \overrightarrow{c}

    \end{bmatrix}$

  2. $0$

  3. $\begin{bmatrix}

    \overrightarrow{a} & \overrightarrow{b} & \overrightarrow{c}

    \end{bmatrix}$

  4. None of these


Correct Option: C
Explanation:

$\left( \overrightarrow{c}+\overrightarrow{b} \right )\times \left ( \overrightarrow{c}+\overrightarrow{a} \right ).\left ( \overrightarrow{c}+\overrightarrow{b}+\overrightarrow{a} \right)$

$= \left(\overrightarrow{c} \times \overrightarrow{a}+\overrightarrow{b} \times \overrightarrow{c} + \overrightarrow{b}\times \overrightarrow{a}\right)\cdot \left ( \overrightarrow{c}+\overrightarrow{b}+\overrightarrow{a} \right)$
$= \overrightarrow{c} \times \overrightarrow{a}\cdot \overrightarrow{c}+\overrightarrow{b} \times \overrightarrow{c}\cdot \overrightarrow{c} + \overrightarrow{b}\times \overrightarrow{a}\cdot \overrightarrow{c}$
$+\overrightarrow{c} \times \overrightarrow{a}\cdot \overrightarrow{b}+\overrightarrow{b} \times \overrightarrow{c}\cdot \overrightarrow{b} + \overrightarrow{b}\times \overrightarrow{a}\cdot \overrightarrow{b}$
$+\overrightarrow{c} \times \overrightarrow{a}\cdot \overrightarrow{a}+\overrightarrow{b} \times \overrightarrow{c}\cdot \overrightarrow{a} + \overrightarrow{b}\times \overrightarrow{a}\cdot \overrightarrow{a}$
$=0+0+ \overrightarrow{b}\times \overrightarrow{a}\cdot \overrightarrow{c}+\overrightarrow{c} \times \overrightarrow{a}\cdot \overrightarrow{b}+0 + 0+0+\overrightarrow{b}\times \overrightarrow{c}\cdot \overrightarrow{a} + 0$
$= \overrightarrow{b}\times \overrightarrow{a}\cdot \overrightarrow{c}+\overrightarrow{c} \times \overrightarrow{a}\cdot \overrightarrow{b}+\overrightarrow{b}\times \overrightarrow{c}\cdot \overrightarrow{a}$

- Hide questions